Convergence Tests: Help Setting Up Problem & Deriving Limit

  • Thread starter Thread starter Badgerspin
  • Start date Start date
  • Tags Tags
    Convergence
Click For Summary
SUMMARY

The discussion revolves around setting up convergence tests in calculus, specifically using the comparison test to derive limits. The user initially struggled to understand the problem setup but later clarified that they successfully identified the limit as (5^(0.6))/(15^6). Despite initially believing this to be incorrect, they ultimately resolved the issue independently. The conversation highlights the importance of understanding convergence tests in mathematical analysis.

PREREQUISITES
  • Understanding of calculus concepts, particularly limits and convergence tests.
  • Familiarity with the comparison test for series convergence.
  • Basic knowledge of mathematical notation and functions.
  • Ability to interpret and analyze mathematical problems from visual aids.
NEXT STEPS
  • Study the comparison test in detail, focusing on its applications in series convergence.
  • Explore other convergence tests such as the ratio test and root test.
  • Practice deriving limits using various convergence tests in calculus.
  • Review common pitfalls in limit calculations to avoid errors in future problems.
USEFUL FOR

Students and educators in mathematics, particularly those studying calculus and series convergence, as well as anyone seeking to improve their problem-solving skills in mathematical analysis.

Badgerspin
Messages
15
Reaction score
0
Generally when I post, it's with a specific problem. However in this case, the issue I'm running into is that I don't even have the slightest idea of how to even setup the problem to attempt it.

Honestly, I can't seem to find this anywhere in my book and I'm not sure what's really being asked of me. Perhaps it's something simple, but I'm not even sure where to start.

Here's a screencap of what I'm being asked to do.
http://i8.photobucket.com/albums/a3/Bivouac/Picture3-2.png?t=1292024780

I'm not looking for solutions (though I won't complain if one comes around), but I would just like to know what's being asked of me, and how to I setup for this kind of problem.

Any help is appreciated.EDIT: I figured it out. Via the comparison test, everything works out. My only issue now is with the last problem in the set. So any help on that one would be very much appreciated.

Via the comparison test, I derived the two highest powers as the limit approached infinity. This came out to be:

(5^(0.6))/(15^6), however it says that I'm wrong in doing this. Perhaps I'm missing something or made a mistake somewhere in the problem.
 
Last edited:
Physics news on Phys.org
Question: A clock's minute hand has length 4 and its hour hand has length 3. What is the distance between the tips at the moment when it is increasing most rapidly?(Putnam Exam Question) Answer: Making assumption that both the hands moves at constant angular velocities, the answer is ## \sqrt{7} .## But don't you think this assumption is somewhat doubtful and wrong?

Similar threads

  • · Replies 4 ·
Replies
4
Views
1K
  • · Replies 16 ·
Replies
16
Views
4K
  • · Replies 8 ·
Replies
8
Views
2K
  • · Replies 14 ·
Replies
14
Views
2K
  • · Replies 11 ·
Replies
11
Views
3K
Replies
2
Views
2K
Replies
4
Views
2K
  • · Replies 4 ·
Replies
4
Views
2K
Replies
4
Views
1K
Replies
3
Views
3K